Difference between revisions of "2022 AMC 10B Problems/Problem 23"
(→Solution) |
(→Solution) |
||
Line 24: | Line 24: | ||
We have | We have | ||
− | < | + | <cmath> |
\begin{align*} | \begin{align*} | ||
P \left( \sum_{n=1}^\tau x_n > 1 \right) | P \left( \sum_{n=1}^\tau x_n > 1 \right) | ||
Line 37: | Line 37: | ||
& = \left( 1 - \frac{1}{2} \right)\left( 1 - \frac{1}{2} \right) | & = \left( 1 - \frac{1}{2} \right)\left( 1 - \frac{1}{2} \right) | ||
+ \left( 1 - \frac{1}{6} \right) \frac{1}{2} \\ | + \left( 1 - \frac{1}{6} \right) \frac{1}{2} \\ | ||
− | & = \boxed{\textbf{(C) }} | + | & = \boxed{\textbf{(C) }} \frac{2}{3}, |
\end{align*} | \end{align*} | ||
− | + | </cmath> | |
where the second equation follows from the property that <math>\left\{ x_n \right\}</math> and <math>\left\{ t_n \right\}</math> are independent sequences, the third equality follows from the lemma above. | where the second equation follows from the property that <math>\left\{ x_n \right\}</math> and <math>\left\{ t_n \right\}</math> are independent sequences, the third equality follows from the lemma above. |
Revision as of 14:12, 17 November 2022
Solution
We use the following lemma to solve this problem.
Let be independent random variables that are uniformly distributed on . Then for ,
For ,
Now, we solve this problem.
We denote by the last step Amelia moves. Thus, . We have
where the second equation follows from the property that and are independent sequences, the third equality follows from the lemma above.
~Steven Chen (Professor Chen Education Palace, www.professorchenedu.com)
Video Solution
~Steven Chen (Professor Chen Education Palace, www.professorchenedu.com)